Probabilidad de la unión de $n$ sucesos

Demostramos una fórmula para hallar la probabilidad de la unión de $n$ sucesos.

    Enunciado
    Sea $(E,\mathcal{M},p)$ un espacio de probabilidad. Se trata de demostrar la fórmula: $$p\left(A_1\cup \ldots \cup A_n\right)=\sum_{i=1}
    ^np\left(A_i\right)-\sum_{i,j=1,\;i < j}
    ^np\left(A_i\cap A_j\right)+$$ $$
    \sum_{i,j,k=1,\;i < j < k}
    ^np\left(A_i\cap A_j\cap A_k\right)+\ldots +(-1)^{n+1}p\left(A_1\cap A_2\cap \ldots \cap A_n\right),$$ para $A_1,\ldots,A_n\in\mathcal{M}$. Se pide:
  1. Demostrar la fórmula para $n=2$.
  2. Demostrar la fórmula para $n=3$.
  3. Usando el método de inducción, demostrar que la fórmula es válida para todo $n\ge 2$.
    Solución

  1. Tenemos $$A_1=A_1\cap E= A_1\cap\left(A_2\cup A_2^c\right)= \left(A_1\cap A_2\right)\cup \left(A_1\cap A_2^c\right),$$ $$A_2=A_2\cap E= A_2\cap\left(A_1\cup A_1^c\right)= \left(A_1\cap A_2\right)\cup \left(A_2\cap A_1^c\right).$$ Además, $A_1$ está expresado como unión de los sucesos incompatibles $A_1\cap A_2$ y $A_1\cap A_2^c$ y por tanto, $$p\left(A_1\right)=p\left(A_1\cap A_2\right)+p\left(A_1\cap A_2^c\right).\qquad [1]$$ De la misma manera, $$p\left(A_2\right)=p\left(A_1\cap A_2\right)+p\left(A_2\cap A_1^c\right).\qquad [2]$$ Por otra parte, $$A_1\cup A_2=\left(A_1\cap A_2\right)\cup \left(A_1\cap A_2^c\right)\cup \left(A_2 \cap A_1^c\right)$$ está expresado como unión de tres sucesos incompatibles y por tanto, $$p\left(A_1\cup A_2\right)=p\left(A_1\cap A_2\right)+p\left(A_1\cap A_2^c\right)+p\left(A_2\cap A_1^c\right).\qquad [3]$$ Sumando miembro a miembro $[1]$ y $[2]$ y pasando un $p\left(A_1\cap A_2\right)$ al primer miembro $$p\left(A_1\right)+p\left(A_2\right)-p\left(A_1\cap A_2\right)=p\left(A_1\cap A_2\right)+p\left(A_1\cap A_2^c\right)+p\left(A_2\cap A_1^c\right),$$ y usando la igualdad $[3]$ queda $$p\left(A_1\cup A_2\right)=p\left(A_1\right)+p\left(A_2\right)-p\left(A_1\cap A_2\right)=\sum_{i=1}^2p\left(A_i\right)+(-1)^{2+1}p\left(A_1\cap A_2\right).$$
  2. Usando la fórmula deducida en el apartado anterior, $$p\left(A_1\cup A_2\cup A_3\right)=p\left[A_1\cup \left(A_2\cup A_3\right)\right]=$$ $$p\left(A_1\right)+p\left(A_2\cup A_3\right)-p\left[A_1\cap\left(A_2\cup A_3\right)\right]$$ $$=p\left(A_1\right)+p\left(A_2\right)+p\left(A_3\right)-p\left(A_2\cap A_3\right)-p\left[\left(A_1\cap A_2\right)\cup \left(A_1\cap A_3\right)\right]$$ $$=p\left(A_1\right)+p\left(A_2\right)+p\left(A_3\right)-p\left(A_2\cap A_3\right)$$ $$-\left[p\left(A_1\cap A_2\right)+p\left(A_1\cap A_3\right)-p\left(\left(A_1\cap A_2\right)\cap \left(A_1\cap A_3\right)\right)\right]$$ $$=p\left(A_1\right)+p\left(A_2\right)+p\left(A_3\right)-p\left(A_2\cap A_3\right)$$ $$-p\left(A_1\cap A_2\right)-p\left(A_1\cap A_3\right)+p\left(A_1\cap A_2\cap A_3\right)$$ $$=\sum_{i=1}^3p\left(A_i\right)-\sum_{i,j=1,\; i < j}^3p\left(A_i\cap A_j\right)+(-1)^{3+1}p\left(A_1\cap A_2\cap A_3\right).$$
  3. Sea la fórmula cierta para $n\ge 3$ y veamos que es cierta para $n+1$: $$p\left(A_1\cup \ldots \cup A_n\cup A_{n+1}\right)=p\left[\left(A_1\cup \ldots \cup A_n\right)\cup A_{n+1}\right]$$ $$=p\left(A_1\cup \ldots \cup A_n\right)+p\left(A_{n+1}\right)-p\left[\left(A_1\cup \ldots \cup A_n\right)\cap A_{n+1}\right]$$ $$=p\left(A_1\cup \ldots \cup A_n\right)+p\left(A_{n+1}\right)-p\left[\left(A_1\cap A_{n+1}\right)\cup \ldots \cup\left(A_n\cap A_{n+1}\right)\right]$$ $$ =\sum_{i=1}
    ^np\left(A_i\right)-\sum_{i,j=1,\;i < j}
    ^np\left(A_i\cap A_j\right)
    + \sum_{i,j,k=1,\;i < j < k}
    ^np\left(A_i\cap A_j\cap A_k\right)$$ $$+\ldots +(-1)^np\left(A_1\cap \ldots \cap A_n\right)+p\left(A_{n+1}\right)$$ $$-\sum_{i=1}
    ^np\left(A_i\cap A_{n+1}\right)+\sum_{i,j=1,\;i < j}
    ^np\left(A_i\cap A_j\cap A_{n+1}\right)$$ $$-\sum_{i,j,k=1,\;i < j < k}
    ^np\left(A_i\cap A_j\cap A_k\cap A_{n+1}\right)$$ $$+\ldots -(-1)^{n+1}p\left(A_1\cap \ldots \cap A_n\cap A_{n+1}\right).$$ Agrupando las probabilidades de las intersecciones de un conjunto, de dos, etc. obtenemos
    $$p\left(A_1\cup \ldots \cup A_n\cup A_{n+1}\right)=\sum_{i=1}
    ^{n+1}p\left(A_i\right)-\sum_{i,j=1,\;i < j}
    ^{n+1}p\left(A_i\cap A_j\right)$$ $$+\sum_{i,j,k=1,\;i < j < k}
    ^{n+1}p\left(A_i\cap A_j\cap A_k\right)+\ldots +(-1)^{(n+1)+1}p\left(A_1\cap\ldots\cap A_n\cap A_{n+1}\right),$$ y la fórmula es cierta para $n+1$.
Esta entrada fue publicada en Miscelánea matemática. Guarda el enlace permanente.